1answer.
Ask question
Login Signup
Ask question
All categories
  • English
  • Mathematics
  • Social Studies
  • Business
  • History
  • Health
  • Geography
  • Biology
  • Physics
  • Chemistry
  • Computers and Technology
  • Arts
  • World Languages
  • Spanish
  • French
  • German
  • Advanced Placement (AP)
  • SAT
  • Medicine
  • Law
  • Engineering
motikmotik
3 years ago
15

Solve: A car travels 2 km North , 10 km East, then 3 km West. pythagorean theorem

Physics
1 answer:
Evgen [1.6K]3 years ago
5 0

________b_____ 7 km east

|

| 2km north.

|a

|

°

pythagorean theorem : ✓a² + b² = c²

c² = a² + b² = 4 + 49 = 53

c = ✓53 km

displacement = c = ✓53 km

distance = 10 + 3 + 2 = 15 km

You might be interested in
A ball rolls 12m in 2.0s. What is the ball’s average velocity?
USPshnik [31]

Answer:

6 m/s

Explanation:

12m / 2s = 6 m/s

Hope that's the answer you seek.

5 0
3 years ago
The school bag of four students A,B,C,D measures 9kg, 2800gm, 2kg and 8000gm respectively. Whose bag is the lightest
inna [77]

Answer:

Student C

Explanation:

order from heaviest to lightest is...

9 kg (A) , 8000g (8 kg) (D) , 2800g (2.8kg) (B), 2 kg (C)

5 0
3 years ago
How many components can be realized of a vector?​
stiv31 [10]

Answer:

<em>two different components</em>

Explanation:

<em>Any two-dimensional vector can be conceived of as having two distinct components. The component of a single vector describes the vector's effect in a specific direction.</em>

3 0
2 years ago
According to Coulomb's law, the electrical force between two charged objects:A.is zero if they are opposite charges.B.increases
kondor19780726 [428]

Answer and explanation:

A correct option is an option (B).

The electrical force between two charges is given as,

F=\frac{1}{4\pi\epsilon_0}\frac{q_1q_1}{r^2}

The electrical force is directly proportional to the product of two charges. Thus Force will depend on two charges irrespective of their signs.

Option (A) is incorrect because if charges are opposite, the value of force will not be zero. It will be -ve.

Option (C) is incorrect because the force is directly proportional to the product of charges, it depends on the amount of charge.

Option (D) is also incorrect because the force in inversly proportional to the distance between two charges. Thus, if the distance between two charges is increased, the force between two charges will decrease.

Concllusion:

The correct option is option (B).

7 0
1 year ago
Which lists the main components of Darwin’s theory of evolution?
Nataliya [291]

Hi!


The correct option is B.


<h3>Explanation</h3>

Evolution is a biological phenomena which descries how the heritable characteristics of a species change over successive generations. These characteristics change as a result of a mutation, natural selection or genetic recombination (to list a few).

Darwin's theory of evolution explained how this phenomena occurs with respect to natural selection. The theory goes on to explain how a species diverges from a common ancestor into two or more different species (meaning different species may have common ancestor). This is a slow process as it can take thousands of years.


Hope this helps!

4 0
3 years ago
Read 2 more answers
Other questions:
  • Describe how evaporation relates to heat regulation in your body
    15·1 answer
  • A 1,400 kg car accelerates from rest to 30 m/s in 6.0 seconds. what is the net force on the car?
    6·1 answer
  • Apply what you know about forces to hypothesize how balanced and unbalanced forces acting on a moving car would affect the motio
    9·1 answer
  • Which action results from the combination of gravity and inertia working on the moon
    10·2 answers
  • A simple generator is used to generate a peak output voltage of 25.0 V . The square armature consists of windings that are 7.0 c
    8·1 answer
  • So we were working on some pulley problems but this one has kinda left me scratching my head, please help! My question is for pa
    10·1 answer
  • Two forces are applied on a body. One produces a force of 480-N directly forward while the other gives a 513-N force at 32.4-deg
    11·1 answer
  • HELP<br> What is the momentum of a 50-kg ice skater gliding across the ice at a speed of 5 m/s?
    5·2 answers
  • Which of the following statements is true?
    7·1 answer
  • When you put water in the ice tray in the freezer, water becomes ice. Ice is more ordered than water. Is this a violation of Sec
    8·1 answer
Add answer
Login
Not registered? Fast signup
Signup
Login Signup
Ask question!